Difference between revisions of "2009 AMC 12B Problems/Problem 14"

(the "add a unit square" solution. :D)
Line 21: Line 21:
  
 
== Solution ==
 
== Solution ==
 
+
=== Solution 1 ===
 
For <math>a\geq 1.5</math> the shaded area is at most <math>1.5</math>, which is too little. Hence <math>a<1.5</math>, and therefore the point <math>(2,1)</math> is indeed inside the shaded part, as shown in the picture.
 
For <math>a\geq 1.5</math> the shaded area is at most <math>1.5</math>, which is too little. Hence <math>a<1.5</math>, and therefore the point <math>(2,1)</math> is indeed inside the shaded part, as shown in the picture.
  
Line 27: Line 27:
  
 
The entire figure has area <math>5</math>, hence we want the shaded part to have area <math>\frac 52</math>. Solving for <math>a</math>, we get <math>a=\boxed{\frac 23}</math>.  The answer is <math>\mathrm{(C)}</math>.
 
The entire figure has area <math>5</math>, hence we want the shaded part to have area <math>\frac 52</math>. Solving for <math>a</math>, we get <math>a=\boxed{\frac 23}</math>.  The answer is <math>\mathrm{(C)}</math>.
 +
 +
=== Solution 2 ===
 +
The total area is 5, so the area of the shaded area is <math>2.5</math>. If we add a unit square in the lower right corner, the area is <math>3.5</math>. Therefore <math>\frac{(3-a)\cdot 3}{2}=3.5</math>, or <math>3-a=\frac{7}{3}</math>. Therefore <math>a=\frac{2}{3}</math>.
  
 
== See Also ==
 
== See Also ==
 
 
{{AMC10 box|year=2009|ab=B|num-b=16|num-a=18}}
 
{{AMC10 box|year=2009|ab=B|num-b=16|num-a=18}}
 
{{AMC12 box|year=2009|ab=B|num-b=13|num-a=15}}
 
{{AMC12 box|year=2009|ab=B|num-b=13|num-a=15}}

Revision as of 12:23, 2 March 2009

The following problem is from both the 2009 AMC 10B #17 and 2009 AMC 12B #14, so both problems redirect to this page.

Problem

Five unit squares are arranged in the coordinate plane as shown, with the lower left corner at the origin. The slanted line, extending from $(a,0)$ to $(3,3)$, divides the entire region into two regions of equal area. What is $a$? [asy] unitsize(1cm); defaultpen(linewidth(.8pt)+fontsize(8pt));  fill((2/3,0)--(3,3)--(3,1)--(2,1)--(2,0)--cycle,gray);  xaxis("$x$",-0.5,4,EndArrow(HookHead,4)); yaxis("$y$",-0.5,4,EndArrow(4));  draw((0,1)--(3,1)--(3,3)--(2,3)--(2,0)); draw((1,0)--(1,2)--(3,2)); draw((2/3,0)--(3,3));  label("$(a,0)$",(2/3,0),S); label("$(3,3)$",(3,3),NE); [/asy]$\textbf{(A)}\ \frac12\qquad \textbf{(B)}\ \frac35\qquad \textbf{(C)}\ \frac23\qquad \textbf{(D)}\ \frac34\qquad \textbf{(E)}\ \frac45$

Solution

Solution 1

For $a\geq 1.5$ the shaded area is at most $1.5$, which is too little. Hence $a<1.5$, and therefore the point $(2,1)$ is indeed inside the shaded part, as shown in the picture.

Then the area of the shaded part is one less than the area of the triangle with vertices $(a,0)$, $(3,0)$, and $(3,3)$. Its area is obviously $\frac{3(3-a)}2$, therefore the area of the shaded part is $\frac{7-3a}{2}$.

The entire figure has area $5$, hence we want the shaded part to have area $\frac 52$. Solving for $a$, we get $a=\boxed{\frac 23}$. The answer is $\mathrm{(C)}$.

Solution 2

The total area is 5, so the area of the shaded area is $2.5$. If we add a unit square in the lower right corner, the area is $3.5$. Therefore $\frac{(3-a)\cdot 3}{2}=3.5$, or $3-a=\frac{7}{3}$. Therefore $a=\frac{2}{3}$.

See Also

2009 AMC 10B (ProblemsAnswer KeyResources)
Preceded by
Problem 16
Followed by
Problem 18
1 2 3 4 5 6 7 8 9 10 11 12 13 14 15 16 17 18 19 20 21 22 23 24 25
All AMC 10 Problems and Solutions
2009 AMC 12B (ProblemsAnswer KeyResources)
Preceded by
Problem 13
Followed by
Problem 15
1 2 3 4 5 6 7 8 9 10 11 12 13 14 15 16 17 18 19 20 21 22 23 24 25
All AMC 12 Problems and Solutions